Jordan Form of Linear Transformation Matrix: R_1[x]

  • Thread starter Thread starter talolard
  • Start date Start date
  • Tags Tags
    Form
Click For Summary
The discussion revolves around finding the Jordan form of a matrix representing a linear transformation on the vector space R_1[x]. The matrix in question is identified as [2 1; 0 1], with eigenvalues 2 and 1, and corresponding eigenvectors {1,0} and {1,-1}. Participants confirm that the matrix is indeed diagonalizable, and by using the eigenvectors to construct matrix P, one can find P^-1AP, which results in a diagonal matrix. The conclusion emphasizes that the original understanding of diagonalizability in relation to Jordan form is correct.
talolard
Messages
119
Reaction score
0

Homework Statement


The question si to find the jordan form of the following, I think it is diagnizable and am wondering if i am misunderstanding something or if diagnizable is a subccase of jordan form:
Anyway
Given the vector space R_1[x] i.e. the space of linear functions (a+bx) find the jordan form of the follwoing matrix
T(a+bx)=2a+b+bx
I thinbk the matrix of T is
[2 1]
[0 1]
Which means it has eigenvalues 2 and 1 with eigenvectors {1,0}. {1,-1}
Is that correct or did I misunderstadn something?
Thanks
Tal


Homework Equations





The Attempt at a Solution

 
Physics news on Phys.org
talolard said:

Homework Statement


The question si to find the jordan form of the following, I think it is diagnizable and am wondering if i am misunderstanding something or if diagnizable is a subccase of jordan form:
Anyway
Given the vector space R_1[x] i.e. the space of linear functions (a+bx) find the jordan form of the follwoing matrix
T(a+bx)=2a+b+bx
I thinbk the matrix of T is
[2 1]
[0 1]
Which means it has eigenvalues 2 and 1 with eigenvectors {1,0}. {1,-1}
Is that correct or did I misunderstadn something?
Thanks
Tal

Everything looks fine, Tal. If you write P using your eigenvectors as columns, and find P-1, P-1AP is a diagonal matrix.
 
Thanks Mark
 
Question: A clock's minute hand has length 4 and its hour hand has length 3. What is the distance between the tips at the moment when it is increasing most rapidly?(Putnam Exam Question) Answer: Making assumption that both the hands moves at constant angular velocities, the answer is ## \sqrt{7} .## But don't you think this assumption is somewhat doubtful and wrong?

Similar threads

  • · Replies 3 ·
Replies
3
Views
2K
  • · Replies 4 ·
Replies
4
Views
1K
  • · Replies 4 ·
Replies
4
Views
2K
  • · Replies 7 ·
Replies
7
Views
4K
  • · Replies 4 ·
Replies
4
Views
2K
  • · Replies 3 ·
Replies
3
Views
2K
  • · Replies 2 ·
Replies
2
Views
2K
  • · Replies 2 ·
Replies
2
Views
2K
  • · Replies 3 ·
Replies
3
Views
2K
  • · Replies 1 ·
Replies
1
Views
2K